Using integrals to prove volume of a pyramid

Click For Summary
The discussion focuses on using integrals to derive the volume of a pyramid, with the user attempting to set up the correct limits for integration. They simplify the problem to one quarter of the pyramid, defining the variables and limits for integration. After performing the integration, the user finds their result does not match the expected volume formula for a pyramid, suggesting an error in their calculations. Other participants point out mistakes in the integration process, particularly in the final steps, indicating that correcting these could yield the correct volume. The conversation emphasizes the importance of careful setup and execution in integral calculus to achieve accurate results.
TromboneNerd
Messages
18
Reaction score
1
the first question i have is if i set it up the iteration right. The I've simplified to one quarter of the pyramid, so the value c is actually 1/2 the base of the whole pyramid. and h is the height of the pyramid. (the pinnacle of the pyramid is along the z axis)

0\leq z\leq -\frac{h}{c}x+h

0\leq y\leq -\frac{c}{h}z+c

0\leq x\leq c

If all that is correct, then ill show my integration work and you can tell me where i went wrong. I don't know how to place lower and upper limits on integrals using Latex, so just use the limits from above.

V=4\int \int \int dydxdz
in respect to y
=4\int \int (-\frac{c}{h} z+c)dzdx
in respect to z
=4\int [-\frac{c}{2h} (-/frac{h}{c} x+h)^{2} +c(-\frac{h}{c} x+h)]dx
finally in respect to x
=4(-\frac{c^{2} h}{6} +ch)

=-\frac{2c^{2} h}{3} +ch)
substitute in .5b for c
=-\frac{b^{2} h}{6} +\frac{bh}{2}

this is obviously not anything close to the formula for the volume of a pyramid, and is completely non sensical. according to this, only very small values of b and h will produce a positive volume. where did i go wrong? I skipped quite a few steps for the sake of typing space, so if your not sure where i got this or that from ill show you the more detailed work where the problem might lie.
 
Physics news on Phys.org
I think you made a mistake with the last integral
 
Yep, you made a mistake
actually if the second term is b^2, you get the answer you need
 
Question: A clock's minute hand has length 4 and its hour hand has length 3. What is the distance between the tips at the moment when it is increasing most rapidly?(Putnam Exam Question) Answer: Making assumption that both the hands moves at constant angular velocities, the answer is ## \sqrt{7} .## But don't you think this assumption is somewhat doubtful and wrong?

Similar threads

  • · Replies 9 ·
Replies
9
Views
2K
  • · Replies 105 ·
4
Replies
105
Views
6K
  • · Replies 14 ·
Replies
14
Views
2K
  • · Replies 2 ·
Replies
2
Views
1K
  • · Replies 1 ·
Replies
1
Views
2K
  • · Replies 7 ·
Replies
7
Views
2K
  • · Replies 2 ·
Replies
2
Views
2K
  • · Replies 4 ·
Replies
4
Views
1K
Replies
11
Views
2K
Replies
6
Views
2K